Suite implicite

Bonjour,
je sèche sur un équivalent dans le cadre d'une suite implicite.

On considère la fonction $f$ définie par $f(x) = \ln^2(x)/x$. On montre facilement que pour tout $n \geq 2$, l'équation $f(x)=1/n$ admet une unique solution $a_n$ dans $[e^2;+\infty[$. Puis, on montre que la suite $(a_n)_{n\geq 2}$ est croissante et diverge vers $+\infty$ avec les méthodes usuelles. Le problème vient quand on en veut un équivalent. Mon exercice fait poser $u_n=a_n/n$ puis admet que $\ln(u_n)=o(\ln(n))$ ce qui permet d'en déduire $u_n \sim \ln^2(n)$ puis un équivalent de $a_n$.

J'aimerais bien savoir comment trouver cette relation $\ln(u_n)=o(\ln(n))$ ? Évidemment, si vous avez une autre démarche pour trouver un équivalent de $u_n$, je suis preneur aussi.

Réponses

  • J'essaierais bien quelque chose comme suit :

    On nous demande de montrer : $\ln(a_n) - \ln(n) = \ln(\frac{a_n}{n}) = o (\ln(n))$.

    En d'autres termes : $\ln(a_n) \sim \ln(n)$.

    Notons $v_n = \ln(a_n)$. Alors $\frac{v_n^2}{\exp(v_n)} = \frac{1}{n}$.

    On note $g(x) = x^2 \cdot \exp(-x)$, pour écrire $g(v_n) = \frac{1}{n}$.

    Maintenant : $g(v_{n+1}) - g(v_n) = \frac{1}{n+1} - \frac{1}{n} = - \frac{1}{n(n+1)}$.

    Or $g'(x) = - (x^2 - 2x)\cdot \exp(-x) \sim -g(x)$. (pour $x\to\infty$)

    Par accroissements finis $g(v_{n+1}) - g(v_n) = g'(c_n) \cdot (v_{n+1}-v_n) \sim -\underbrace{g(c_n)}_{\sim 1/n} (v_{n+1}-v_n)$.

    Il nous reste donc $- \frac{1}{n} \cdot (v_{n+1}-v_n) \sim - \frac{1}{n(n+1)}$.

    En sommant l'équivalent $v_{n+1} - v_n \sim \frac{1}{n+1}$, il vient bien : $v_n \sim \ln(n)$.
  • L'équation s'écrit $\ln^2(nu_n)=u_n$, ce qui donne $\ln n=\sqrt{u_n}-\ln u_n$. Comme $(u_n)$ diverge vers l'infini, $\ln u_n=o\bigl(\sqrt{u_n}\bigr)$, d'où $\ln n\sim \sqrt{u_n}$, puis $\ln u_n=o(\ln n)$.
  • Bonjour,

    J'aurais procédé ainsi:

    Par définition de $(a_n)_{n\geqslant 2}$, $\ln(a_n)^2=\dfrac{a_n}{n}$.
    Comme $(a_n)_{n\geqslant 2}$ diverge vers $+\infty$, on en déduit que $\left(\dfrac{a_n}{n}\right)_{n\geqslant 2}$ diverge également vers $+\infty$.
    On pose, pour tout $n\geqslant 2$, $u_n = \dfrac{a_n}{n}$.
    On montre alors sans difficulté que, pour tout $n\geqslant 2$, $nu_n = e^{\sqrt{u_n}}$
    Ainsi, pour tout $n\geqslant 2$, $\ln(n)+\ln(u_n) = \sqrt{u_n}$
    Donc, pour tout $n\geqslant 2$, $\dfrac{\ln(n)}{\sqrt{u_n}}+\dfrac{\ln(u_n)}{\sqrt{u_n}} = 1$
    Comme $(u_n)_{n\geqslant 2}$ diverge vers $+\infty$ (d'après ce qui précède), par croissances comparées, $\lim\limits_{n\to +\infty} \dfrac{\ln(u_n)}{\sqrt{u_n}} = 0$
    Donc, $\lim\limits_{n\to +\infty} \dfrac{\ln(n)}{\sqrt{u_n}}=1$
    Donc, $\sqrt{u_n}\underset{n\to +\infty}{\sim}\ln(n)$
    Donc, $u_n\underset{n\to +\infty}{\sim}\ln(n)^2$
    Donc, $$a_n\underset{n\to +\infty}{\sim}n\ln(n)^2$$

    Edit: c'est exactement ce que propose Math Coss
  • Bonjour,

    pour MathCoss et Bbidule, comment faire pour prouver que $u_n$ diverge vers $+ \infty$ ? Il faut que je vérifie mon calcul, mais il me semble que $f(n ln(n)) > 1/n$ et comme $f$ est décroissante, on obtient $n ln(n) < a_n$.

    La méthode de marsup fonctionne bien, mais je n'aurais pas pensé à utiliser ça !
  • Par exemple, ou bien comme l'a dit Bbidule : $u_n=\ln^2a_n$ et $(a_n)$ diverge vers l'infini.
Connectez-vous ou Inscrivez-vous pour répondre.